Shelbyville’s New Dealership

Welcome! You are encouraged to register with the site and login (for free). When you register, you support the site and your question history is saved.

A luxury car dealership in Shelbyville is expected to close due to competition from an Alexa dealership, which also sells luxury cars. Nevertheless, the demand for cars in Shelbyville will be sufficient for a new dealership to succeed as a new business. After all, in previous years, the Dealeo discount car dealership in Shelbyville has driven several other dealerships out of business, but in every case there has been sufficient demand for a new dealership to enter the city and be successful for a time.

Which of the following, if true, most seriously weakens the argument?

Review: Shelbyville's New Dealership


Explanation

Reading the question: we have three dealerships that are compared in a roundabout fashion. To establish which ideas are connected and which things are compared, we can use term matching:



There are a couple of points of technique that we can highlight here. First, if an argument does not easily break up into terms, you can make some progress by simply imagining (or jotting down, if necessary) entire clauses on both sides of the table. Second, the conjunctions "nevertheless" and "but" indicate that the clause that comes before each one is neither evidence nor conclusion; it's something the argument is disputing. Matching terms further, we see that the terms of the evidence and the conclusion match well. The question really boils down to whether this case is like "every other case." Do any answer choices suggest this case is different? We can use that as our basic relevance filter.

Applying the filter: (B) and (D) are closest to our expectation. Choice (A) is immaterial to the "successful for a time" condition and is out. We also knock out (C) as there is no contrast in time or in dealerships. Choice (D) would strengthen the argument, not weaken it. We're down to (B) and (E). Both of them require us to get specific about luxury vs. discount. Fact #1 of the argument is that a luxury dealer knocked out a luxury dealer. We don't know whether there is demand for more than one luxury dealer. Answer choice (B) indicates that demand for luxury cars is down, so it weakens the idea that there is room for another one after Alexa. (B) looks good. Choice (E) could either weaken or strengthen the argument. It would suggest that there is plenty of demand other than luxury, which would strengthen the argument. Or it could suggest that non-luxury demand is already covered. But (B) objectively weakens the argument and therefore is the correct answer. The correct answer is (B).


If you believe you have found an error in this question or explanation, please contact us and include the question title or URL in your message.